Difference between revisions of "2013 USAMO Problems/Problem 1"
m (→Solution 1) |
(Added a second solution) |
||
Line 83: | Line 83: | ||
Firstly, it is easy to see by that <math>\omega_A, \omega_B, \omega_C</math> concur at a point <math>M</math>. Let <math>XM</math> meet <math>\omega_B, \omega_C</math> again at <math>D</math> and <math>E</math>, respectively. Then by Power of a Point, we have <cmath>XM \cdot XE = XZ \cdot XP \quad\text{and}\quad XM \cdot XD = XY \cdot XP</cmath> Thusly <cmath>\frac{XY}{XZ} = \frac{XD}{XE}</cmath> But we claim that <math>\triangle XDP \sim \triangle PBM</math>. Indeed, <cmath>\measuredangle XDP = \measuredangle MDP = \measuredangle MBP = - \measuredangle PBM</cmath> and <cmath>\measuredangle DXP = \measuredangle MXY = \measuredangle MXA = \measuredangle MRA = \measuredangle MRB = \measuredangle MPB = -\measuredangle BPM</cmath> | Firstly, it is easy to see by that <math>\omega_A, \omega_B, \omega_C</math> concur at a point <math>M</math>. Let <math>XM</math> meet <math>\omega_B, \omega_C</math> again at <math>D</math> and <math>E</math>, respectively. Then by Power of a Point, we have <cmath>XM \cdot XE = XZ \cdot XP \quad\text{and}\quad XM \cdot XD = XY \cdot XP</cmath> Thusly <cmath>\frac{XY}{XZ} = \frac{XD}{XE}</cmath> But we claim that <math>\triangle XDP \sim \triangle PBM</math>. Indeed, <cmath>\measuredangle XDP = \measuredangle MDP = \measuredangle MBP = - \measuredangle PBM</cmath> and <cmath>\measuredangle DXP = \measuredangle MXY = \measuredangle MXA = \measuredangle MRA = \measuredangle MRB = \measuredangle MPB = -\measuredangle BPM</cmath> | ||
Therefore, <math>\frac{XD}{XP} = \frac{PB}{PM}</math>. Analogously we find that <math>\frac{XE}{XP} = \frac{PC}{PM}</math> and we are done. | Therefore, <math>\frac{XD}{XP} = \frac{PB}{PM}</math>. Analogously we find that <math>\frac{XE}{XP} = \frac{PC}{PM}</math> and we are done. | ||
+ | |||
courtesy v_enhance | courtesy v_enhance | ||
+ | ---- | ||
+ | ==Solution 2== | ||
+ | |||
+ | [https://www.flickr.com/photos/127013945@N03/14800492500/lightbox/ Diagram] | ||
+ | Refer to the Diagram link. | ||
+ | |||
+ | By Miquel's Theorem, there exists a point at which <math>\omega_A, \omega_B, \omega_C</math> intersect. We denote this point by <math>M.</math> Now, we angle chase: | ||
+ | <cmath>\angle YMX = 180^{\circ} - \angle YXM - \angle XYM</cmath><cmath>= 180^{\circ} - \angle AXM - \angle PYM</cmath><cmath>= \left(180^{\circ} - \angle ARM\right) - \angle PRM</cmath><cmath>= \angle BRM - \angle PRM</cmath><cmath>= \angle BRP = \angle BMP.</cmath> | ||
+ | In addition, we have | ||
+ | <cmath>\angle ZMX = 180^{\circ} - \angle MZY - \angle ZYM - \angle YMX</cmath><cmath>= 180^{\circ} - \angle MZP - \angle PYM - \angle BMP</cmath><cmath>= 180^{\circ} - \angle MCP - \angle PBM - \angle BMP</cmath><cmath>= \left(180^{\circ} - \angle PBM - \angle BMP\right) - \angle MCP</cmath><cmath>= \angle BPM - \angle MCP</cmath><cmath>= 180^{\circ} - \angle MPC - \angle MCP</cmath><cmath>= \angle CMP.</cmath> | ||
+ | Now, by the Ratio Lemma, we have | ||
+ | <cmath>\frac{XY}{XZ} = \frac{MY}{MZ} \cdot \frac{\sin \angle YMX}{\sin \angle ZMX}</cmath><cmath>= \frac{\sin \angle YZM}{\sin \angle ZYM} \cdot \frac{\sin \angle BMP}{\sin \angle CMP}</cmath> (by the Law of Sines in <math>\triangle MZY</math>)<cmath>= \frac{\sin \angle PZM}{\sin \angle PYM} \cdot \frac{\sin \angle BMP}{\sin \angle CMP}</cmath><cmath>= \frac{\sin \angle PCM}{\sin \angle PBM} \cdot \frac{\sin \angle BMP}{\sin \angle CMP}</cmath><cmath>= \frac{MB}{MC} \cdot \frac{\sin \angle BMP}{\sin \angle CMP}</cmath> (by the Law of Sines in <math>\triangle MBC</math>)<cmath>= \frac{PB}{PC}</cmath> by the Ratio Lemma. | ||
+ | The proof is complete. | ||
+ | |||
{{MAA Notice}} | {{MAA Notice}} |
Revision as of 01:56, 21 August 2014
Problem
In triangle , points lie on sides respectively. Let , , denote the circumcircles of triangles , , , respectively. Given the fact that segment intersects , , again at respectively, prove that
Solution 1
In this solution, all lengths and angles are directed.
Firstly, it is easy to see by that concur at a point . Let meet again at and , respectively. Then by Power of a Point, we have Thusly But we claim that . Indeed, and Therefore, . Analogously we find that and we are done.
courtesy v_enhance
Solution 2
Diagram Refer to the Diagram link.
By Miquel's Theorem, there exists a point at which intersect. We denote this point by Now, we angle chase: In addition, we have Now, by the Ratio Lemma, we have (by the Law of Sines in ) (by the Law of Sines in ) by the Ratio Lemma. The proof is complete.
The problems on this page are copyrighted by the Mathematical Association of America's American Mathematics Competitions.